Jump to content

Photo

Chưng minh các BĐT sau:


  • Please log in to reply
4 replies to this topic

#1
lymiu

lymiu

    Binh nhất

  • Thành viên
  • 38 posts

Với a,b,c,d là các số không âm. Chứng minh các BĐT sau: :wacko: :wacko: :wacko:

 

1~     Cho: $\frac{1}{1+a}+\frac{1}{1+b}+\frac{1}{1+c}+\frac{1}{1+d}= 3$

         CMR:      $abcd\leq \frac{1}{81}$

 

 

2~    $\left ( 1+a \right )\left ( 1+b \right )\left ( 1+c \right )\geq \left ( 1+\sqrt[3]{abc} \right )^{3}$

 

 

3~    $\left ( a^{2} +b^{2}+c^{2}\right )\left ( \frac{1}{a+b}+\frac{1}{b+c} +\frac{1}{c+a}\right )\geq \frac{3}{2}\left ( a+b+c \right )$


                   (~~) Hãy để mỗi ngày của bạn thật sự có ý nhja (~~) 

                        :botay  **==  = :)  :luoi  :oto:  **==  :luoi:  :ukliam2:  :ukliam2:  :ukliam2: 

                                                                                  https://www.facebook.com/ly.miu.589


#2
Nguyen Duc Phu

Nguyen Duc Phu

    Trung sĩ

  • Thành viên
  • 184 posts

Với a,b,c,d là các số không âm. Chứng minh các BĐT sau: :wacko: :wacko: :wacko:

 

1~     Cho: $\frac{1}{1+a}+\frac{1}{1+b}+\frac{1}{1+c}+\frac{1}{1+d}= 3$

         CMR:      $abcd\leq \frac{1}{81}$

 

 

2~    $\left ( 1+a \right )\left ( 1+b \right )\left ( 1+c \right )\geq \left ( 1+\sqrt[3]{abc} \right )^{3}$

 

 

3~    $\left ( a^{2} +b^{2}+c^{2}\right )\left ( \frac{1}{a+b}+\frac{1}{b+c} +\frac{1}{c+a}\right )\geq \frac{3}{2}\left ( a+b+c \right )$

1) Từ giả thiết suy ra

$\frac{b}{b+1}+\frac{c}{c+1}+\frac{d}{d+1}\leq 1-\frac{a}{a+1}=\frac{1}{a+1}$

Áp dụng bất đẳng thức AM-GM cho 3 số không âm ta có

$\frac{1}{a+1}\geq \frac{b}{b+1}+\frac{c}{c+1}+\frac{d}{d+1}\geq 3\sqrt[3]{\frac{bcd}{(b+1)(c+1)(d+1)}}$

Tương tự

$\frac{1}{b+1}\geq \frac{a}{a+1}+\frac{c}{c+1}+\frac{d}{d+1}\geq 3\sqrt[3]{\frac{acd}{(a+1)(c+1)(d+1)}}$

$\frac{1}{c+1}\geq \frac{a}{a+1}+\frac{b}{b+1}+\frac{d}{d+1}\geq 3\sqrt[3]{\frac{abd}{(a+1)(b+1)(d+1)}}$

$\frac{1}{d+1}\geq \frac{a}{a+1}+\frac{b}{b+1}+\frac{c}{c+1}\geq 3\sqrt[3]{\frac{abc}{(b+1)(c+1)(a+1)}}$

Nhân từng vế bốn bất đẳng thức, ta được $1\geq 81abcd$ nên $abcd\leq \frac{1}{81}$


Edited by Nguyen Duc Phu, 31-01-2015 - 23:03.

Khi chúng ta dựa vào mày tính làm trung gian cho sự hiểu biết về thế giới thì trí thông minh của chúng ta đã trở thành trí tuệ giả tạo.(Nicholas  Carr trong Trí tuệ giả tạo-Internet đã làm gì chúng ta?)

 


#3
Nguyen Duc Phu

Nguyen Duc Phu

    Trung sĩ

  • Thành viên
  • 184 posts

2)BĐT tương đương:

$a+b+c+ab+bc+ac\geq 3(\sqrt[3]{abc})^2+3\sqrt[3]{abc}$

Áp dụng bđt AM-GM cho 3 số không âm, ta có:

$a+b+c\geq 3\sqrt[3]{abc}$

$ab+bc+ca\geq 3(\sqrt[3]{abc})^2$

Suy ra điều phải chứng minh.


Khi chúng ta dựa vào mày tính làm trung gian cho sự hiểu biết về thế giới thì trí thông minh của chúng ta đã trở thành trí tuệ giả tạo.(Nicholas  Carr trong Trí tuệ giả tạo-Internet đã làm gì chúng ta?)

 


#4
Hoang Long Le

Hoang Long Le

    Thượng sĩ

  • Thành viên
  • 270 posts

Với a,b,c,d là các số không âm. Chứng minh các BĐT sau: :wacko: :wacko: :wacko:

 

 

3~    $\left ( a^{2} +b^{2}+c^{2}\right )\left ( \frac{1}{a+b}+\frac{1}{b+c} +\frac{1}{c+a}\right )\geq \frac{3}{2}\left ( a+b+c \right )$

Sử dụng AM-GM và Cauchy-Schwarz 

$VT\geq \frac{1}{3}(a+b+c)^2.(\frac{1}{a+b}+\frac{1}{b+c} +\frac{1}{c+a})=\frac{1}{6}(a+b+c)[(a+b)+(b+c)+(c+a)](\frac{1}{a+b}+\frac{1}{b+c} +\frac{1}{c+a})\geq \frac{9}{6}(a+b+c)=\frac{3}{2}(a+b+c)$



#5
dogsteven

dogsteven

    Đại úy

  • Thành viên
  • 1567 posts

Bài 2. Áp dụng bất đẳng thức AM-GM:

$\dfrac{1}{1+a}+\dfrac{1}{1+b}+\dfrac{1}{1+c} \geqslant \dfrac{3}{\sqrt[3]{(1+a)(1+b)(1+c)}}$

$\dfrac{a}{1+a}+\dfrac{b}{1+b}+\dfrac{c}{1+c} \geqslant \dfrac{3\sqrt[3]{abc}}{\sqrt[3]{(1+a)(1+b)(1+c)}}$

$\Rightarrow 3\geqslant \dfrac{3(1+\sqrt[3]{abc})}{\sqrt[3]{(1+a)(1+b)(1+c)}} \Leftrightarrow (1+a)(1+b)(1+c) \geqslant (1+\sqrt[3]{abc})^3$


Quyết tâm off dài dài cày hình, số, tổ, rời rạc.





1 user(s) are reading this topic

0 members, 1 guests, 0 anonymous users